Preptest #30 Section #2 Question #15 Forum

Prepare for the LSAT or discuss it with others in this forum.
Post Reply
User avatar
CincinnatusND

Bronze
Posts: 134
Joined: Fri Jan 06, 2012 9:54 pm

Preptest #30 Section #2 Question #15

Post by CincinnatusND » Wed May 07, 2014 5:29 pm

Can someone help me understand the reasoning behind the correct response on this one?


It seems to me that the correct response (a) wouldn't be a necessary assumption, as the stem only states that the banks will lend more if the standards are relaxed, not that the banks will lend as much as before the down-turn if they are relaxed.

If the banks did lose a significant amount of money to fund deposits, why couldn't the amount they lend nevertheless increase after a relaxations of regulatory practices?

Am I mis-interpreting the stem somehow? I find it to be generally a fairly flawed argument. Maybe that is contributing to my confusion here.

Thanks in advance for any help you can provide!

User avatar
SpiritofFire

New
Posts: 48
Joined: Thu Dec 05, 2013 11:48 am

Re: Preptest #30 Section #2 Question #15

Post by SpiritofFire » Wed May 07, 2014 6:45 pm

It's necessary since, if it did cause a significant decrease in funds, that could be a strong reason for believing that banks still won't lend money even if regulations were loosened. It would throw the whole argument into jeopardy.

Another way to look at this is to see the flaw's essence: correlation = causation. The flaw ignores other possible things that could cause the effect ( in this case, less funds to work with)

User avatar
Jeffort

Gold
Posts: 1888
Joined: Wed Jun 18, 2008 4:43 pm

Re: Preptest #30 Section #2 Question #15

Post by Jeffort » Wed May 07, 2014 9:15 pm

CincinnatusND wrote:
If the banks did lose a significant amount of money to fund deposits, why couldn't the amount they lend nevertheless increase after a relaxations of regulatory practices?
The banks can't loan out $$ they don't have. If less people deposited $$ with the bank or deposited less $$ into an account with the bank, the bank has less $$ on hand they could loan out to others. Make sense? The banks have to have $$ on hand to loan it to other people. If you ask a friend that is broke to loan you $$, (s)he can't cuz doesn't have any cash. Gotta have $$ on hand to be able to loan $$ out to people. Can't let somebody borrow something from you that you don't have.

rmcmanus

New
Posts: 13
Joined: Wed May 07, 2014 1:24 am

Re: Preptest #30 Section #2 Question #15

Post by rmcmanus » Wed May 07, 2014 10:47 pm

"during the recent eco downturn, banks contributed to the decline of loaning LESS money" -> answer A) "the downturn did NOT cause a SIGNIFICANT decrease)

User avatar
CincinnatusND

Bronze
Posts: 134
Joined: Fri Jan 06, 2012 9:54 pm

Re: Preptest #30 Section #2 Question #15

Post by CincinnatusND » Wed May 07, 2014 10:53 pm

Ah thanks everyone, for whatever reason I got it in my head that it was a question on what assumption the argument required, and didn't realize it was just asking what the argument assumed. So the flawed reasoning really threw me off.

Well, now I know to look more closely at not only the stem, but at the actual question as well, when none of the ac's seem correct.

Want to continue reading?

Register now to search topics and post comments!

Absolutely FREE!


rmcmanus

New
Posts: 13
Joined: Wed May 07, 2014 1:24 am

Re: Preptest #30 Section #2 Question #15

Post by rmcmanus » Wed May 07, 2014 10:55 pm

I actually took pt 30 today myself and missed that question also.

User avatar
Christine (MLSAT)

Bronze
Posts: 357
Joined: Fri Nov 22, 2013 3:41 pm

Re: Preptest #30 Section #2 Question #15

Post by Christine (MLSAT) » Wed May 07, 2014 11:00 pm

CincinnatusND wrote:Ah thanks everyone, for whatever reason I got it in my head that it was a question on what assumption the argument required, and didn't realize it was just asking what the argument assumed. So the flawed reasoning really threw me off.

Well, now I know to look more closely at not only the stem, but at the actual question as well, when none of the ac's seem correct.
I'm a little confused - your bolded is the language of necessary assumption, and this question is absolutely a necessary assumption question.

Or did you mean that you mistook it for asking what's also required to be true, i.e., an inference question?

User avatar
CincinnatusND

Bronze
Posts: 134
Joined: Fri Jan 06, 2012 9:54 pm

Re: Preptest #30 Section #2 Question #15

Post by CincinnatusND » Wed May 07, 2014 11:18 pm

Christine (MLSAT) wrote:
CincinnatusND wrote:Ah thanks everyone, for whatever reason I got it in my head that it was a question on what assumption the argument required, and didn't realize it was just asking what the argument assumed. So the flawed reasoning really threw me off.

Well, now I know to look more closely at not only the stem, but at the actual question as well, when none of the ac's seem correct.
I'm a little confused - your bolded is the language of necessary assumption, and this question is absolutely a necessary assumption question.

Or did you mean that you mistook it for asking what's also required to be true, i.e., an inference question?
Sorry, I don't really know the exact categories of questions.

I guess I'm still not getting it then. Maybe it is just that the argument seems so broken to me that none of the AC's really seem to validate it?

I'm use to questions where choosing a necessary assumption really seems to complete the chain of reasoning.

User avatar
Christine (MLSAT)

Bronze
Posts: 357
Joined: Fri Nov 22, 2013 3:41 pm

Re: Preptest #30 Section #2 Question #15

Post by Christine (MLSAT) » Wed May 07, 2014 11:22 pm

CincinnatusND wrote:
Christine (MLSAT) wrote:
CincinnatusND wrote:Ah thanks everyone, for whatever reason I got it in my head that it was a question on what assumption the argument required, and didn't realize it was just asking what the argument assumed. So the flawed reasoning really threw me off.

Well, now I know to look more closely at not only the stem, but at the actual question as well, when none of the ac's seem correct.
I'm a little confused - your bolded is the language of necessary assumption, and this question is absolutely a necessary assumption question.

Or did you mean that you mistook it for asking what's also required to be true, i.e., an inference question?
Sorry, I don't really know the exact categories of questions.

I guess I'm still not getting it then. Maybe it is just that the argument seems so broken to me that none of the AC's really seem to validate it?

I'm use to questions where choosing a necessary assumption really seems to complete the chain of reasoning.
Ahhhh, I see the problem!

Okay, so, a 'necessary assumption' is does NOT have to 'complete the chain of reasoning.' SUFFICIENT assumptions will complete the chain of reasoning.

Necessary assumptions are things we NEED for the argument to even have a chance at working. When we take those things away, the argument dies (because we NEED them). Sufficient assumptions, on the other hand, are things that are by themselves SUFFICIENT, or ENOUGH to GUARANTEE that the conclusion will follow.

These are two totally different thought processes, and some assumptions can be both necessary *and* sufficient (so, these are not mutually exclusive categories).

Does that help a little?

User avatar
CincinnatusND

Bronze
Posts: 134
Joined: Fri Jan 06, 2012 9:54 pm

Re: Preptest #30 Section #2 Question #15

Post by CincinnatusND » Wed May 07, 2014 11:31 pm

Christine (MLSAT) wrote:
CincinnatusND wrote:
Christine (MLSAT) wrote:
CincinnatusND wrote:Ah thanks everyone, for whatever reason I got it in my head that it was a question on what assumption the argument required, and didn't realize it was just asking what the argument assumed. So the flawed reasoning really threw me off.

Well, now I know to look more closely at not only the stem, but at the actual question as well, when none of the ac's seem correct.
I'm a little confused - your bolded is the language of necessary assumption, and this question is absolutely a necessary assumption question.

Or did you mean that you mistook it for asking what's also required to be true, i.e., an inference question?
Sorry, I don't really know the exact categories of questions.

I guess I'm still not getting it then. Maybe it is just that the argument seems so broken to me that none of the AC's really seem to validate it?

I'm use to questions where choosing a necessary assumption really seems to complete the chain of reasoning.
Ahhhh, I see the problem!

Okay, so, a 'necessary assumption' is does NOT have to 'complete the chain of reasoning.' SUFFICIENT assumptions will complete the chain of reasoning.

Necessary assumptions are things we NEED for the argument to even have a chance at working. When we take those things away, the argument dies (because we NEED them). Sufficient assumptions, on the other hand, are things that are by themselves SUFFICIENT, or ENOUGH to GUARANTEE that the conclusion will follow.

These are two totally different thought processes, and some assumptions can be both necessary *and* sufficient (so, these are not mutually exclusive categories).

Does that help a little?
Yeah, sorry, I originally mistook it as looking for a 'sufficient assumption' instead of a 'necessary assumption.'

Thanks for helping me clear this up!

Now I feel like I should really study the different types of questions in depth, but I'm at the point where I'm missing 0-2 questions per section and I'm worried that'll lead to me over-thinking things even more.

Want to continue reading?

Register for access!

Did I mention it was FREE ?


Post Reply

Return to “LSAT Prep and Discussion Forum”